proof-pile / books /napkin /cardinal.tex
zhangir-azerbayev
added books
afd65d6
raw
history blame
18 kB
\chapter{Cardinals}
An ordinal measures a total ordering.
However, it does not do a fantastic job at measuring size.
For example, there is a bijection between the elements of $\omega$ and $\omega+1$:
\[
\begin{array}{rccccccc}
\omega+1 = & \{ & \omega & 0 & 1 & 2 & \dots & \} \\
\omega = & \{ & 0 & 1 & 2 & 3 & \dots & \}.
\end{array}
\]
In fact, as you likely already know,
there is even a bijection between $\omega$ and $\omega^2$:
\[
\begin{array}{l|cccccc}
+ & 0 & 1 & 2 & 3 & 4 & \dots \\ \hline
0 & 0 & 1 & 3 & 6 & 10 & \dots \\
\omega & 2 & 4 & 7 & 11 & \dots & \\
\omega \cdot 2 & 5 & 8 & 12 & \dots & & \\
\omega \cdot 3 & 9 & 13 & \dots & & & \\
\omega \cdot 4 & 14 & \dots & & & &
\end{array}
\]
So ordinals do not do a good job of keeping track of size.
For this, we turn to the notion of a cardinal number.
\section{Equinumerous sets and cardinals}
\begin{definition}
Two sets $A$ and $B$ are \vocab{equinumerous}, written $A \approx B$,
if there is a bijection between them.
\end{definition}
\begin{definition}
A \vocab{cardinal} is an ordinal $\kappa$ such that
for no $\alpha < \kappa$ do we have $\alpha \approx \kappa$.
\end{definition}
\begin{example}[Examples of cardinals]
Every finite number is a cardinal.
Moreover, $\omega$ is a cardinal.
However, $\omega+1$, $\omega^2$, $\omega^{2015}$ are not,
because they are countable.
\end{example}
\begin{example}[$\omega^\omega$ is countable]
Even $\omega^\omega$ is not a cardinal,
since it is a countable union
\[ \omega^\omega = \bigcup_n \omega^n \]
and each $\omega^n$ is countable.
\end{example}
\begin{ques}
Why must an infinite cardinal be a limit ordinal?
\end{ques}
\begin{remark}
There is something fishy about the definition of a cardinal:
it relies on an \emph{external} function $f$.
That is, to verify $\kappa$ is a cardinal I can't just look at $\kappa$ itself;
I need to examine the entire universe $V$ to make sure
there does not exist a bijection $f : \kappa \to \alpha$ for $\alpha < \kappa$.
For now this is no issue, but later in model theory
this will lead to some highly counterintuitive behavior.
\end{remark}
\section{Cardinalities}
Now that we have defined a cardinal, we can discuss the size
of a set by linking it to a cardinal.
\begin{definition}
The \vocab{cardinality} of a set $X$
is the \emph{least} ordinal $\kappa$ such that $X \approx \kappa$.
We denote it by $\left\lvert X \right\rvert$.
\end{definition}
\begin{ques}
Why must $\left\lvert X \right\rvert$ be a cardinal?
\end{ques}
\begin{remark}
One needs the well-ordering theorem (equivalently, choice)
in order to establish that such an ordinal $\kappa$ actually exists.
\end{remark}
Since cardinals are ordinals, it makes sense to ask whether $\kappa_1 \le \kappa_2$,
and so on.
Our usual intuition works well here.
\begin{proposition}[Restatement of cardinality properties]
Let $X$ and $Y$ be sets.
\begin{enumerate}[(i)]
\ii $X \approx Y$ if and only $\left\lvert X \right\rvert = \left\lvert Y \right\rvert$,
if and only if there's a bijection from $X$ to $Y$.
\ii $\left\lvert X \right\rvert \le \left\lvert Y \right\rvert$
if and only if there is an injective map $X \injto Y$.
\end{enumerate}
\end{proposition}
Diligent readers are invited to try and prove this.
\section{Aleph numbers}
\prototype{$\aleph_0 = \omega$, and $\aleph_1$ is the first uncountable ordinal.}
First, let us check that cardinals can get arbitrarily large:
\begin{proposition}
We have $\left\lvert X \right\rvert < \left\lvert \PP(X) \right\rvert$ for every set $X$.
\end{proposition}
\begin{proof}
There is an injective map $X \injto \PP(X)$
but there is no injective map $\PP(X) \injto X$ by \Cref{lem:cantor_diag}.
\end{proof}
Thus we can define:
\begin{definition}
For a cardinal $\kappa$, we define $\kappa^+$ to be the least cardinal above $\kappa$,
called the \vocab{successor cardinal}.
\end{definition}
This $\kappa^+$ exists and has $\kappa^+ \le \left\lvert \PP(\kappa) \right\rvert$.
Next, we claim that:
\begin{exercise}
Show that if $A$ is a set of cardinals, then $\cup A$ is a cardinal.
\end{exercise}
Thus by transfinite induction we obtain that:
\begin{definition}
For any $\alpha \in \On$, we define the \vocab{aleph numbers} as
\begin{align*}
\aleph_0 &= \omega \\
\aleph_{\alpha+1} &= \left( \aleph_\alpha \right)^+ \\
\aleph_{\lambda} &= \bigcup_{\alpha < \lambda} \aleph_\alpha.
\end{align*}
\end{definition}
Thus we have the sequence of cardinals
\[
0 < 1 < 2 < \dots < \aleph_0 < \aleph_1 < \dots < \aleph_\omega < \aleph_{\omega+1} < \dots.
\]
By definition, $\aleph_0$ is the cardinality of the natural numbers,
$\aleph_1$ is the first uncountable ordinal, \dots.
We claim the aleph numbers constitute all the cardinals:
\begin{lemma}[Aleph numbers constitute all infinite cardinals]
If $\kappa$ is a cardinal then
either $\kappa$ is finite (i.e.\ $\kappa \in \omega$) or
$\kappa = \aleph_\alpha$ for some $\alpha \in \On$.
\end{lemma}
\begin{proof}
Assume $\kappa$ is infinite, and take $\alpha$ minimal with $\aleph_\alpha \ge \kappa$.
Suppose for contradiction that we have $\aleph_\alpha > \kappa$.
We may assume $\alpha > 0$, since the case $\alpha = 0$ is trivial.
If $\alpha = \ol\alpha + 1$ is a successor, then
\[ \aleph_{\ol\alpha} < \kappa < \aleph_{\alpha}
= (\aleph_{\ol\alpha})^+ \]
which contradicts the definition of the successor cardinal.
If $\alpha = \lambda$ is a limit ordinal, then $\aleph_\lambda$ is the
supremum $\bigcup_{\gamma < \lambda} \aleph_\gamma$.
So there must be some $\gamma < \lambda$ with $\aleph_\gamma > \kappa$,
which contradicts the minimality of $\alpha$.
\end{proof}
\begin{definition}
An infinite cardinal which is not a successor cardinal
is called a \vocab{limit cardinal}.
It is exactly those cardinals of the form $\aleph_\lambda$,
for $\lambda$ a limit ordinal, plus $\aleph_0$.
\end{definition}
\section{Cardinal arithmetic}
\prototype{$\aleph_0 \cdot \aleph_0 = \aleph_0 + \aleph_0 = \aleph_0$}
Recall the way we set up ordinal arithmetic.
Note that in particular, $\omega + \omega > \omega$ and $\omega^2 > \omega$.
Since cardinals count size, this property is undesirable, and
we want to have
\begin{align*}
\aleph_0 + \aleph_0 &= \aleph_0 \\
\aleph_0 \cdot \aleph_0 &= \aleph_0
\end{align*}
because $\omega + \omega$ and $\omega \cdot \omega$ are countable.
In the case of cardinals, we simply ``ignore order''.
The definition of cardinal arithmetic is as expected:
\begin{definition}[Cardinal arithmetic]
Given cardinals $\kappa$ and $\mu$, define
\[ \kappa + \mu
\defeq
\left\lvert
\left( \left\{ 0 \right\} \times \kappa \right)
\cup
\left( \left\{ 1 \right\} \times \mu \right)
\right\rvert
\]
and
\[
\kappa \cdot \mu
\defeq
\left\lvert \mu \times \kappa \right\rvert
.
\]
\end{definition}
\begin{ques}
Check this agrees with what you learned in pre-school
for finite cardinals.
\end{ques}
\begin{abuse}
This is a slight abuse of notation since we are using
the same symbols as for ordinal arithmetic,
even though the results are different ($\omega \cdot \omega = \omega^2$
but $\aleph_0 \cdot \aleph_0 = \aleph_0$).
In general, I'll make it abundantly clear whether I am talking
about cardinal arithmetic or ordinal arithmetic.
\end{abuse}
To help combat this confusion, we use separate symbols for ordinals and cardinals.
Specifically, $\omega$ will always refer to $\{0,1,\dots\}$ viewed as an ordinal;
$\aleph_0$ will always refer to the same set viewed as a cardinal.
More generally,
\begin{definition}
Let $\omega_\alpha = \aleph_\alpha$ viewed as an ordinal.
\end{definition}
However, as we've seen already we have that $\aleph_0 \cdot \aleph_0 = \aleph_0$.
In fact, this holds even more generally:
\begin{theorem}[Infinite cardinals squared]
Let $\kappa$ be an infinite cardinal.
Then $\kappa \cdot \kappa = \kappa$.
\end{theorem}
\begin{proof}
Obviously $\kappa \cdot \kappa \ge \kappa$,
so we want to show $\kappa \cdot \kappa \le \kappa$.
The idea is to try to repeat the same proof
that we had for $\aleph_0 \cdot \aleph_0 = \aleph_0$,
so we re-iterate it here. We took the ``square'' of
elements of $\aleph_0$, and then
\emph{re-ordered} it according to the diagonal:
\[
\begin{array}{l|cccccc}
& 0 & 1 & 2 & 3 & 4 & \dots \\ \hline
0 & 0 & 1 & 3 & 6 & 10 & \dots \\
1 & 2 & 4 & 7 & 11 & \dots & \\
2 & 5 & 8 & 12 & \dots & & \\
3 & 9 & 13 & \dots & & & \\
4 & 14 & \dots & & & &
\end{array}
\]
We'd like to copy this idea for a general $\kappa$;
however, since addition is less well-behaved for infinite ordinals
it will be more convenient to use $\max\{\alpha,\beta\}$
rather than $\alpha+\beta$.
Specifically, we put the ordering $<_{\text{max}}$
on $\kappa \times \kappa$ as follows:
for $(\alpha_1, \beta_1)$ and $(\alpha_2, \beta_2)$ in $\kappa \times \kappa$
we declare $(\alpha_1, \beta_1) <_{\text{max}} (\alpha_2, \beta_2)$ if
\begin{itemize}
\ii $\max \left\{ \alpha_1, \beta_1 \right\} < \max \left\{ \alpha_2, \beta_2 \right\}$ or
\ii $\max \left\{ \alpha_1, \beta_1 \right\} = \max \left\{ \alpha_2, \beta_2 \right\}$ and $(\alpha_1, \beta_1)$
is lexicographically earlier than $(\alpha_2, \beta_2)$.
\end{itemize}
This alternate ordering (which deliberately avoids referring
to the addition) looks like:
\[
\begin{array}{l|cccccc}
& 0 & 1 & 2 & 3 & 4 & \dots \\ \hline
0 & 0 & 1 & 4 & 9 & 16 & \dots \\
1 & 2 & 3 & 5 & 10 & 17 & \dots \\
2 & 6 & 7 & 8 & 11 & 18 & \dots \\
3 & 12 & 13 & 14 & 15 & 19 & \dots \\
4 & 20 & 21 & 22 & 23 & 24 & \dots \\
\vdots & \vdots & \vdots & \vdots & \vdots & \vdots & \ddots \\
\end{array}
\]
Now we proceed by transfinite induction on $\kappa$.
The base case is $\kappa = \aleph_0$, done above.
Now, $<_{\text{max}}$ is a well-ordering of $\kappa \times \kappa$,
so we know it is in order-preserving bijection with some ordinal $\gamma$.
Our goal is to show that $\left\lvert \gamma \right\rvert \le \kappa$.
To do so, it suffices to prove that for any $\ol\gamma \in \gamma$,
we have $\left\lvert \ol\gamma \right\rvert < \kappa$.
Suppose $\ol\gamma$ corresponds to the point $(\alpha, \beta) \in \kappa \times \kappa$
under this bijection.
If $\alpha$ and $\beta$ are both finite
then certainly $\ol\gamma$ is finite too.
Otherwise, let $\ol\kappa = \max \{\alpha, \beta\} < \kappa$;
then the number of points below $\ol\gamma$ is at most
\[
\left\lvert \alpha \right\rvert \cdot \left\lvert \beta \right\rvert
\le \ol\kappa \cdot \ol\kappa
= \ol\kappa
\]
by the inductive hypothesis.
So $\left\lvert \ol\gamma \right\rvert \le \ol\kappa < \kappa$ as desired.
\end{proof}
From this it follows that cardinal addition and multiplication is really boring:
\begin{theorem}[Infinite cardinal arithmetic is trivial]
Given cardinals $\kappa$ and $\mu$,
one of which is infinite, we have
\[ \kappa \cdot \mu = \kappa + \mu
= \max\left\{ \kappa, \mu \right\}.\]
\end{theorem}
\begin{proof}
The point is that both of these are less than the square of the maximum.
Writing out the details:
\begin{align*}
\max \left\{ \kappa, \mu \right\}
&\le \kappa + \mu \\
&\le \kappa \cdot \mu \\
&\le \max \left\{ \kappa, \mu \right\}
\cdot \max \left\{ \kappa, \mu \right\} \\
&= \max\left\{ \kappa, \mu \right\}. \qedhere
\end{align*}
\end{proof}
\section{Cardinal exponentiation}
\prototype{$2^\kappa = \left\lvert \PP(\kappa) \right\rvert$.}
\begin{definition}
Suppose $\kappa$ and $\lambda$ are cardinals.
Then
\[ \kappa^\lambda
\defeq \left\lvert \mathscr F(\lambda, \kappa) \right\rvert.
\]
Here $\mathscr F(A,B)$ is the set of functions from $A$ to $B$.
\end{definition}
\begin{abuse}
As before, we are using the same notation for
both cardinal and ordinal arithmetic. Sorry!
\end{abuse}
In particular, $2^\kappa = \left\lvert \PP(\kappa) \right\rvert > \kappa$,
and so from now on we can use the notation $2^\kappa$ freely.
(Note that this is totally different from ordinal arithmetic;
there we had $2^\omega = \bigcup_{n\in\omega} 2^n = \omega$.
In cardinal arithmetic $2^{\aleph_0} > \aleph_0$.)
I have unfortunately not told you what $2^{\aleph_0}$ equals.
A natural conjecture is that $2^{\aleph_0} = \aleph_1$; this is called the
\vocab{Continuum Hypothesis}.
It turns out that this is \emph{undecidable} -- it is not possible
to prove or disprove this from the $\ZFC$ axioms.
\section{Cofinality}
\prototype{$\aleph_0$, $\aleph_1$, \dots\ are all regular, but $\aleph_\omega$ has cofinality $\omega$.}
\begin{definition}
Let $\lambda$ be an ordinal (usually a limit ordinal),
and $\alpha$ another ordinal.
A map $f : \alpha \to \lambda$ of ordinals is called \vocab{cofinal}
if for every $\ol\lambda < \lambda$, there is some $\ol\alpha \in \alpha$
such that $f(\ol\alpha) \ge \ol\lambda$.
In other words, the map reaches arbitrarily high into $\lambda$.
\end{definition}
\begin{example}
[Example of a cofinal map]
\listhack
\begin{enumerate}[(a)]
\ii The map $\omega \to \omega^\omega$ by $n \mapsto \omega^n$ is cofinal.
\ii For any ordinal $\alpha$, the identity map $\alpha \to \alpha$ is cofinal.
\end{enumerate}
\end{example}
\begin{definition}
Let $\lambda$ be a limit ordinal.
The \vocab{cofinality} of $\lambda$, denoted $\cof(\lambda)$,
is the smallest ordinal $\alpha$ such that there is a cofinal map
$\alpha \to \lambda$.
\end{definition}
\begin{ques}
Why must $\alpha$ be an infinite cardinal?
\end{ques}
Usually, we are interested in taking the cofinality of a cardinal $\kappa$.
Pictorially, you can imagine standing at the bottom of the universe and looking
up the chain of ordinals to $\kappa$.
You have a machine gun and are firing bullets upwards, and you want to get arbitrarily
high but less than $\kappa$.
The cofinality is then the number of bullets you need to do this.
We now observe that ``most'' of the time, the cofinality of a cardinal is itself.
Such a cardinal is called \vocab{regular}.
\begin{example}[$\aleph_0$ is regular]
$\cof(\aleph_0) = \aleph_0$, because no finite subset of
$\aleph_ 0 = \omega$ can reach arbitrarily high.
\end{example}
\begin{example}[$\aleph_1$ is regular]
$\cof(\aleph_1) = \aleph_1$.
Indeed, assume for contradiction that some countable
set of ordinals $A = \{ \alpha_0, \alpha_1, \dots \} \subseteq \aleph_1$
reaches arbitrarily high inside $\aleph_1$.
Then $\Lambda = \cup A$ is a \emph{countable} ordinal,
because it is a countable union of countable ordinals.
In other words $\Lambda \in \aleph_1$.
But $\Lambda$ is an upper bound for $A$, contradiction.
\end{example}
On the other hand, there \emph{are} cardinals which are not regular;
since these are the ``rare'' cases we call them \vocab{singular}.
\begin{example}[$\aleph_\omega$ is not regular]
Notice that $\aleph_0 < \aleph_1 < \aleph_2 < \dots$ reaches
arbitrarily high in $\aleph_\omega$, despite only having $\aleph_0$ terms.
It follows that $\cof(\aleph_\omega) = \aleph_0$.
\end{example}
We now confirm a suspicion you may have:
\begin{theorem}
[Successor cardinals are regular]
If $\kappa = \ol\kappa^+$ is a successor cardinal,
then it is regular.
\end{theorem}
\begin{proof}
We copy the proof that $\aleph_1$ was regular.
Assume for contradiction that for some $\mu \le \ol\kappa$,
there are $\mu$ sets reaching arbitrarily high in $\kappa$ as a cardinal.
Observe that each of these sets must have cardinality at most $\ol\kappa$.
We take the union of all $\mu$ sets, which gives an ordinal $\Lambda$
serving as an upper bound.
The number of elements in the union is at most
\[ \#\text{sets} \cdot \#\text{elms}
\le \mu \cdot \ol\kappa = \ol\kappa \]
and hence $\left\lvert \Lambda \right\rvert \le \ol\kappa < \kappa$.
\end{proof}
\section{Inaccessible cardinals}
So, what about limit cardinals?
It seems to be that most of them are singular: if $\aleph_\lambda \ne \aleph_0$ is a limit ordinal,
then the sequence $\{\aleph_\alpha\}_{\alpha \in \lambda}$ (of length $\lambda$) is certainly cofinal.
\begin{example}[Beth fixed point]
Consider the monstrous cardinal
\[ \kappa = \aleph_{\aleph_{\aleph_{\ddots}}}. \]
This might look frighteningly huge, as $\kappa = \aleph_\kappa$,
but its cofinality is $\omega$ as it is the limit of the sequence
\[ \aleph_0, \aleph_{\aleph_0}, \aleph_{\aleph_{\aleph_0}}, \dots \]
\end{example}
More generally, one can in fact prove that
\[ \cof(\aleph_\lambda) = \cof(\lambda). \]
But it is actually conceivable that $\lambda$ is so large
that $\left\lvert \lambda \right\rvert = \left\lvert \aleph_\lambda \right\rvert$.
A regular limit cardinal other than $\aleph_0$ has a special name: it is \vocab{weakly inaccessible}.
Such cardinals are so large that it is impossible to prove or disprove their existence in $\ZFC$.
It is the first of many so-called ``large cardinals''.
An infinite cardinal $\kappa$ is a strong limit cardinal if
\[ \forall \ol\kappa < \kappa \quad 2^{\ol\kappa} < \kappa \]
for any cardinal $\ol\kappa$. For example, $\aleph_0$ is a strong limit cardinal.
\begin{ques}
Why must strong limit cardinals actually be limit cardinals?
(This is offensively easy.)
\end{ques}
A regular strong limit cardinal other than $\aleph_0$
is called \vocab{strongly inaccessible}.
\section\problemhead
\begin{problem}
Compute $\left\lvert V_\omega \right\rvert$.
\begin{hint}
$\sup_{k \in \omega} \left\lvert V_k \right\rvert$.
\end{hint}
\end{problem}
\begin{problem}
Prove that for any limit ordinal $\alpha$, $\cof(\alpha)$ is a \emph{regular} cardinal.
\begin{hint}
Rearrange the cofinal maps to be nondecreasing.
\end{hint}
\end{problem}
\begin{sproblem}
[Strongly inaccessible cardinals]
\label{prob:strongly_inaccessible}
Show that for any strongly inaccessible $\kappa$,
we have $\left\lvert V_\kappa \right\rvert = \kappa$.
\end{sproblem}
\begin{problem}
[K\"onig's theorem]
Show that \[ \kappa^{\cof(\kappa)} > \kappa \] for every infinite cardinal $\kappa$.
\end{problem}